Kaplan Daily Questions
A 55-year-old male presents complaining of "difficulty writing" using his dominant hand and some "slurred" speech. He has a h/o hypertension, DM type II, and hypertriglyceridemia. Which of the following would you anticipate to find on a CT scan of his head, as the explanation for his chief complaint? A. hemorrhage in the distribution of his posterior cerebral artery B. hemorrhage in the distribution of the middle cerebral artery C. hypodensity measuring 12 mm by 21 mm in the distribution of the posterior cerebral artery D. hypodensity measuring 4 mm by 4 mm in the internal capsule E. calcifications bilaterally, in the third ventricles
Answer D Hypodensity measuring 4 mm by 4 mm in the internal capsule The main clue in this question is that most strokes are ischemic - especially with this patient's history (ruling out hemorrhagic infarct - A & B). Calcifications (E) are not indicative of stroke at all. When trying to decide between C & D - the size of the area needs to correlate with the patient's sxs. If the patient had a HUGE infarct (12 x 21) he would have significantly more signs and symptoms than just some slurred speech and difficulty writing. (In addition, as it turns out, the internal capsule is supplied by the MCA - which is the most common vessel involved in an ischemic stroke)
A patient warrants antihypertensive medication use for the duration of her pregnancy. Which of the following is the antihypertensive recommended for such patients? A. alpha-methyldopa B. captopril C. nifedipine D. propranolol E. clonidine
Answer: A Alpha-methyldopa Methyldopa (Aldomet) is indicated in pregnancy. ACEI's (captopril) are contraindicated in pregnancy. ß Blockers can be used but may cause growth restriction. Diuretics are not used in pregnancy. Calcium channel blockers and centrally acting agents (while not contraindicated) should be avoided.
This papulosquamous eruption is most common in young adults. A single oval patch is generally noted several days before a more generalized, fawn-colored rash erupts. This rash is most prevalent on the trunk, and the proximal upper and lower extremities. The rash spontaneously disappears over five to six weeks. The diagnosis is: A. pityriasis rosea B. tinea corporis C. psoriasis D. atopic dermatitis E. sporotrichosis
Answer: A Pityriasis rosea This is the classic presentation of pityriasis rosea. Remember that the differential diagnosis includes secondary syphilis, so if there is a positive sexual history - consider ordering a VDRL/RPR. The "herald" patch which is described is pathognomonic for pityriasis.
A 42-year-old female presents after finding a firm, painless bump in her right eyelid. On examination, you note a 6 mm mass within the tarsus of the right eye. The skin is freely movable over the mass. The remainder of the ophthalmoscopic examination is unremarkable. Which of the following is the most likely diagnosis? A. Pterygium B. Chalazion C. Ectropion D. External hordeolum E. Internal hordeolum
Answer: B Chalazion A chalazion is a painless chronic mass in the eyelid. Hordeolum are acute and red and painful. Pterygium involves the sclera. Ectropion is when the eyelid sags outwardly and the lid doesn't close well.
A 27-year-old nulliparous female presents because she's been trying to get pregnant for two years, but has failed. She relates a history of a misdiagnosis of appendicitis that lead to abscess formation when she was 14 years old. Which of the following diagnostic studies would be most helpful at this point in her evaluation? A. TSH level B. hysterosalpingogram C. laparoscopy D. PAP smear E. pelvic ultrasound
Answer: B Hysterosalpingogram While I would disagree that an invasive procedure like HSG should be done first-line, the thing to remember in this question is that the patient has reason to have tubal scarring from adhesions (and there is no better answer listed to choose), so, for a board exam I would choose this answer. The TSH level would not be indicated (she has not had a pg loss), lap could diagnose the tubal scarring but would be done after an abnormal HSG. Pap smear is screening for cervical cancer and not indicated in this case of infertility, and the pelvic US would yield nothing diagnostically about the tubes.
Which term is used to describe the characteristic concave or "spoon-shaped" nails of iron deficiency anemia? A. leukonychia B. koilonychias C. clubbing D. onycholysisE. paronychia
Answer: B Koilonychia Koilonychia is a spoon-shaping of the nail itself. It is usually a result of iron deficiency anemia. Leukonychia is often associated with hypoalbuminemia that causes partial or complete white discoloration of the nails. Leukonychia may also appear as a rare side effect of systemic chemotherapy in some oncological patients but may also be present with arsenic poisoning, renal failure pneumonia, or heart disease. Clubbing of the nails is an actual thickening or elevation of the nail bed - it is a sign of a release of TNF associated with pulmonary disorders (tissue necrosis factor) typically found in bronchiectasis, lung cancers and cystic fibrosis (the nails are NOT necessarily cyanotic.) Onycholysis is a painless separation of the nail from the nail bed. Several or all nails are usually affected - there are many causes. Paronychia is an infection of the nail bed and nail margin, usually from trauma or more commonly, nail-biting.
Combinations of antimicrobial agents are commonly employed in the treatment of meningitis in infants less than three months old. Ampicillin is commonly an agent included in this regimen. Ampicillin is used empirically for the possible presence of: A. Escherichia coli B. Listeria monocytogenes C. Cytomegalovirus D. Herpesvirus E. Hemophilus influenzae
Answer: B Listeria monocytogenes While E. Coli can infect an infant delivered vaginally, Ampicillin is given to eliminate Listeria. CMV & HSV are viruses. E. Coli & H. Flu are also typically resistant to ampicillin, and more likely a broader spectrum antibiotic would be used.
A patient presents complaining of generalized swelling. Her urine is positive for protein. Her serum testing reveals hyperlipidemia and hypoalbuminemia. Which of the following is the most likely diagnosis? A. cirrhosis B. nephrotic syndrome C. congestive heart failure D. cystitits E. pyelonephritis
Answer: B Nephrotic syndrome This is a classic presentation of nephrotic syndrome.
A patient describes a history of recurrent bouts of uveitis. Her chemistry panel reveals elevated serum calcium and uric acid levels. Her anergy screen is negative. Her chest x-ray demonstrates bilateral hilar adenopathy. Which diagnosis is most likely? A. Silicosis B. Sarcoidosis C. Alpha-1 antitrypsin deficiency D. Histoplasmosis E. Tuberculosis
Answer: B Sarcoidosis Sarcoidosis typically presents with hilar lymphadenopathy and noncaseating granulomas of the lungs (and other organs). In addition, patients may have eye involvement (uveitis). Elevations of ACE, Calcium and uric acid are frequently seen.
A 22-year-old patient was involved in an automobile accident and is comatose. Which of the following diagnostic modalities would be least useful in this patient's evaluation? A. CT scan B. skull radiographs C. MRI scan D. EEGE. PET scan
Answer: B Skull radiographs Of these choices, the skull film is LEAST useful. Whether or not the skull is fractured, any LOC in a head trauma requires imaging for bleeding. CT would be the imaging test of choice. MRI should be done for a more chronic bleed. EEG would help to establish brain activity. PET scan (while not done often) would establish physiologic function (uptake of glucose). A positive or negative plain film of the skull would supply NO FURTHER INFORMATION.
A 34-year-old female patient presents to the ER with sharp pleuritic-type chest pain. An x-ray shows an enlarged cardiac silhouette suggestive of pericarditis. On more careful questioning, the patient admits to a polyarticular arthritis involving the small joints of her hands. Her obstetrical history is positive for two third trimester spontaneous abortions. At this point, you would suspect which autoimmune process as the cause of her symptoms: A. Rheumatoid arthritis B. Systemic Lupus Erythematosis C. Wegener's granulomatosis D. Sjogren's SyndromeE. Scleroderma
Answer: B Systemic Lupus Erythematosus 40-60% of patients with SLE have serositis (pleuritis and pericarditis). That, combined with joint pains, habitual AB - should cause you to order an ANA.
Which of the following is NOT a characteristic feature of nephrotic syndrome? A. proteinuria B. hematuria C. hypoalbuminemia D. hyperlipidemia E. generalized edema
Answer: B. Hematuria Hematuria is present in NEPHRITIC syndrome. Nephrotic syndrome characteristically includes proteinuria (>3.5 gm/day), with resulting low serum albumin, hyperlipidemia, hypertension, hypercoagulability, and generalized edema (from oncotic third-spacing)
A 48-year-old nurse with a body mass index of 31 presents for an evaluation of back pain. She relates that historically, she had a positive PPD test a year ago and did not follow-up as directed. She has recently been experiencing night sweats and coughing. An x-ray of her lumbar spine reveals osteopenia and cortical breakdown of vertebral bodies L4 and L5. Which of the following diagnosis is most suspect? A. compression fractures secondary to obesity B. degenerative joint disease C. Potts disease D. compression fractures secondary to osteoporosisE. spondylolisthesis
Answer: C Pott's disease Pott's disease is TB of the spine. She could have any of the other diseases, but the question states that she had a positive PPD and did not take meds...that is the clue that she has active TB in the spine.
52-year-old male presents complaining of urinary frequency, with hesitancy, and nocturia for the past few months. During his physical examination, you note a nontender, non-enlarged prostate with an isolated right posterior lobe nodule. Which of the following options is most appropriate? A. Order a serum acid phosphatase level B. Initiate prazosin and schedule a follow-up appointment in 6 weeks C. Refer the patient for an ultrasound of the prostate and order a PSA level D. Reassure the patient and schedule a follow-up appointment in six months E. Initiate norfloxacin therapy for 7 days and schedule follow-up in two weeks
Answer: C Refer the patient for an ultrasound of the prostate and order a PSA level This patient has an isolated nodule of the prostate gland — cancer until proven otherwise. You should order an ultrasound and a PSA. BPH will present as a diffuse enlargement, and not a discrete nodule.
While suturing a wound, you opt to use lidocaine with epinephrine. The rationale for your choice is: A. an increase in the absorption of the lidocaine. B. an increase in the diffusion of the lidocaine into the nerve's myelin sheath. C. an increase in the blood flow to the area of injection. D. an increase in the duration of anesthesia. E. a decrease in the risk of infection at the site of injection.
Answer: D An increase in the duration of anesthesia Lidocaine with epinephrine both increases the duration of anesthesia and decreases blood flow to the area of injection-i.e. penis, nose, fingers, toes
Which of the following thyroid profiles is most compatible with a diagnosis of primary hypothyroidism? A. a low TSH (thyroid stimulating hormone) level and a high T4 B. a low TSH level and a normal T4 C. a low TSH level and a low T4 D. a high TSH level and a low T4 E. a high TSH and a high T4
Answer: D High TSH level and a low T4 Low T4 is diagnostic for low thyroid function. If the pituitary is normal (as in primary thyroid disease) the TSH should be high as the pituitary tries to stimulate the failing thyroid gland.
A 6-year-old child falls onto his right arm. An x-ray demonstrates a buckle in the cortices of the distal radius, proximal to the growth plate, without angulation. What is the term used to describe this fracture A. Salter-Harris Type III B. Salter Harris Type IV C. Salter Harris Type V D. torus E. greenstick
Answer: D Torus fracture Torus or buckle fracture is most common in a child. This is proximal to the epiphyseal plate and so is not a Salter-Harris issue. Greenstick fracture is also common in children in long bones and is a fracture which "bends" the bone without fracturing it.
An EKG demonstrates a PR interval of 0.16 seconds, a P to QRS relationship of 1:1, a variable heart rate and an R to R interval that is noted to accelerate ad decelerate during the respiratory cycle. What is the diagnosis? A. Wenckebach B. third degree heart block C. atrial fibrillation D. sinus arrhythmia E. atrial flutter
Answer: D sinus arrhythmia This is sinus arrhythmia. Wenckebach & third-degree AVB would have a nonconducted P wave. A-fib would have no P waves and an irregularly irregular rhythm. A flutter would have "flutter waves" or a regular rhythm of 150.
All of the following are factors that predispose a patient to the development of gastroesophageal reflux EXCEPT: A. hiatal hernia B. pregnancy C. scleroderma D. an incompetent esophageal sphincter E. pernicious anemia
Answer: E Pernicious anemia Pernicious anemia has no correlation with GERD. It is an autoimmune destruction of the gastric parietal cells that make intrinsic factor. Signs & symptoms are not present until B12 levels are very low (and include peripheral neuropathies & ataxia)
A patient is experiencing an acute exacerbation of asthma. Which of the following drugs would be the least useful in the management of this asthma attack? A. albuterol B. cromolyn sodium C. prednisone D. theophylline E. epinephrine
Cromolyn sodium Cromolyn is a mast cell stabilizer and must be present in the system PRIOR to the symptoms. It would be useless in a patient already having symptoms.
A 44-year-old woman has a 10-year history of progressive dysphagia without pain. The dysphagia affects all kinds of food, without preference for solids. The food sticks in the xiphoid area and can be helped along by drinking large amounts of water and sitting up straight until the whole bolus passes into the stomach. She also describes many episodes of regurgitation of foul-smelling but undigested food, either when she leans forward or when she is asleep at night. She used to be heavier and lost weight as the disease progressed. A barium swallow shows a massively dilated proximal esophagus with a narrow, beak-like appearance in the lower sphincteric area. Which of the following is the most likely original pathophysiology that explains the development of this problem? A. Failure of the lower esophageal sphincter to relax B. Reflux of acid gastric juice into the lower esophagus C. Replacement of smooth muscle by fibrous tissue D. Spasm of the lower esophageal sphincter E. Weak or nonexistent peristaltic activity in the upper esophagus
The correct answer is A. The clinical and radiologic picture is that of achalasia, which starts with failure of the lower esophageal sphincter to relax. The word "achalasia" in fact means "failure to relax." In addition to having inability of the lower sphincter to relax, patients also have poor tertiary stripping waves (peristalsis) in the esophagus, which causes an inability of the patient to pass the food from the esophagus into the stomach. Because this is a motor disorder, esophageal manometry could also be used to make this diagnosis. Acid reflux (choice B) patients would have a long history of heartburn and pain leading eventually to stricture. Patients who have acid reflux would have retained food in the esophagus with dysphagia, but this retained food would be the result of a stricture and this would be readily identified on the barium swallow or esophageal manometry studies. Replacement of smooth muscle by fibrous tissue (choice C) is the pathophysiologic process with scleroderma. Other manifestations of that disease would include induration of the skin, arthralgias, and Raynaud phenomenon. As noted, the initial problem is failure of the lower esophageal sphincter to relax. It is not spasm of the lower esophageal sphincter (choice D) , as some might imagine, or a primary lack of propulsive motility (choice E) . Achalasia is an esophageal motor disorder that is characterized by the failure of the lower esophageal sphincter to relax and also by the loss of tertiary stripping waves in the lower esophagus. Because it is a motor disorder, the patient will have dysphagia for both solids and liquids from the onset of the condition. This disease can slowly progress. Endoscopy, barium swallow, and esophageal manometry can all be used in the evaluation of this condition. Definitive therapy is with pneumatic dilation or laparoscopic cardiomyotomy.
A 58-year-old woman is found on a routine medical check-up to have a serum calcium level of 11.8 mg/dL, where the upper limit of normal is 9.5 mg/dL. Repeated determinations confirm values between 10.9 and 12.2 mg/dL, and she is found to have elevated concentrations of parathyroid hormone. She is asymptomatic, has no pertinent family history, and has no evidence of renal stones or bone disease. She is offered the option of elective parathyroidectomy, but she declines and elects to have close medical follow-up. In addition to monitoring serum calcium levels, which of the following therapeutics should be given? A. Pamidronate (Aredia) B. Long-term calcitonin C. Low calcium intake D. Low-dose thiazides E. Vitamin D analogues
The correct answer is A. The underlying pathology is most likely a single parathyroid adenoma, for which surgical removal is the only cure. In postmenopausal women, however, bisphosphonate therapy can be given to inhibit osteoclastic activity, which reduces bone resorption and turnover. Hydration is also an essential treatment for this condition to help decrease serum calcium levels. Calcitonin (choice B) is useful when dealing acutely with extremely high levels of serum calcium, but long-term therapy is not routinely pursued. Resistance to its hypocalcemic effects develops within 1 or 2 days. It may have a role in the management of symptomatic osteoporotic compression fracture management, because this agent has some bone anesthesia associated with its use. The excessive cost of the oral agent make calcitonin a less commonly used agent. Low calcium intake (choice C) can result in increased hormone production and can accentuate bone disease. Calcium intake for this woman should be adequate, but not low. Thiazides (choice D) are contraindicated in the setting of hypercalcemia because they can worsen hypercalcemia (when diuretics are used to control excessive hypercalcemia, furosemide is recommended). Vitamin D analogues (choice E) are used in patients who have hyperparathyroidism secondary to renal failure. Vitamin D analogs can increase the absorption of calcium, which potentially could worsen a patient who has hyperparathyroidism who already has an elevated serum calcium level. PEARL: Patients who have hypercalcemia caused by primary hyperparathyroidism need to have serum calcium levels monitored so that if they rise to 15 mg/dL or higher, intervention can take place. When patients have hyperparathyroidism and conservative therapy is elected to pursue, adequate hydration states are essential to maintain. Definitive therapy with surgery to remove the parathyroid hormone can be performed if medical therapy fails to control the hypercalcemia. In addition to hydration therapy, bisphosphonates may be used to inhibit osteoclastic activity and reduce bone resorption. Calcitonin may be used as adjunctive short-term therapy, but its effectiveness in hypercalcemia management in patients who have primary hyperparathyroidism is short-lived and a very expensive alternative treatment.
One week after having an upper respiratory infection, a man develops conjunctival hyperemia, watery discharge, and ocular irritation. Although both eyes are involved by the time a health care provider is consulted, the symptoms began in only one eye. On the morning of the health care appointment, the man has trouble opening his eyelids on awakening, as they are "glued shut." Physical examination demonstrates hyperemic bulbar and tarsal conjunctivae. No purulent fluid is seen. The preauricular lymph node on one side is enlarged. Which of the following pathogens is most likely to cause these symptoms? A. Adenovirus B. Herpes simplex I C. Herpes simplex II D. Neisseria gonorrhoeae E. Staphylococcus aureus
The correct answer is A. This is viral conjunctivitis, an acute conjunctival inflammation most often caused by adenovirus. Most patients with this condition have had a recent exposure to someone with viral conjunctivitis or have had a recent viral upper respiratory infection. The presentation illustrated here is typical. Severe cases additionally have photophobia and a foreign body sensation. Pseudomembranes of fibrin and inflammatory cells on the conjunctival surfaces can occur. Corneal involvement can leave residual scarring visible by slit lamp for up to 2 years after a severe case. Cases are self-limited but tend to be extremely contagious, so care should be taken to have both the patient and health care provider wash their hands very thoroughly after touching the face. Herpes virus (choices B and C) can cause corneal ulceration, hyperkeratosis, or scarring. It is commonly identified on staining of the eye, which reveals a dendritic corneal lesion. Neisseria gonorrhoeae (choice D) can cause conjunctivitis in both adults and neonates. It produces a purulent, rather than a watery discharge. Staphylococcus aureus (choice E) is a common cause of bacterial conjunctivitis. It produces a purulent, rather than a watery discharge. PEARL: Viral conjunctivitis is a benign self-limiting condition. The most common causative organism is adenovirus. Viral conjunctivitis typically is associated with a watery discharge. Both bacterial and viral conjunctivitis may cause a matting of the eyelids together. Preauricular lymph node involvement is much more commonly seen with viral conjunctivitis. Adenoviral conjunctivitis is the typical cause of epidemic keratoconjunctivitis and follicular conjunctivitis. Viral conjunctivitis is more commonly seen in children. It is highly contagious and spreads by both direct contact with people or by touching contaminated surfaces that contain these secretions.
A 33-year-old woman comes to the healthcare provider because of anxiety and irritability. She states that approximately 10 days before the onset of her menses, she goes from "feeling fine" to "feeling awful." She has frequent angry outbursts toward her husband and children, feels depressed and irritable, and cries easily. She also develops severe breast tenderness during this time. The symptoms resolve soon after the start of her menses and she feels fine until the symptoms start over again as the next menses approach. She has had this constellation of symptoms every cycle over the past year and feels that she can't go on like this anymore, as her job and family relations are suffering greatly. She has no other medical problems and takes no medications. Her physical examination is entirely normal. Which of the following is the most appropriate management? Fluoxetine (Prozac) Oophorectomy Primrose oil Progesterone Vitamin B6 (pyridoxine)
The correct answer is A. This patient has a presentation that is consistent with premenstrual dysphoric disorder (PMDD). This disorder is described in the Diagnostic and Statistical Manual of Mental Disorders, and is considered a form of premenstrual syndrome (PMS) with more severe emotional symptoms. Selective serotonin reuptake inhibitors (SSRIs) are the initial drugs of choice for PMDD or severe PMS. Fluoxetine (Prozac) and sertraline (Zoloft) are both approved for the management of this disorder. Patients who have severe PMS may be placed on a daily dose. Another approach that has been shown to be effective is intermittent therapy, in which the SSRI is administered only during the symptomatic phase. This method reduces the overall rate of side effects and is less expensive. Fluoxetine (Prozac) has the longest half-life of all the SSRIs, thus may be especially helpful in this disorder as missed doses are not as impactful. Oophorectomy (choice B) as a treatment for PMS is controversial because the surgery is irreversible, there is morbidity and mortality associated with the surgery, and the resulting loss of hormones is also associated with health risks. The most appropriate management for this patient is fluoxetine. Primrose oil (choice C) has been studied as a treatment for the symptoms of PMS and has not been found to be effective. There may be some benefit for the use of this agent in treating the soreness of the breasts that this patient is experiencing. Like primrose oil, progesterone (choice D) also has been evaluated as a possible treatment for PMS. In several trials it was not shown to be more effective than placebo, although there is some evidence that it may help for specific symptoms such as breast tenderness and bloating. Vitamin B6 (choice E) is considered to be of limited benefit for the treatment of PMS. High doses (>100 mg/day) are associated with peripheral neuropathy. PEARL: Premenstrual dysphoric disorder (PMDD) is associated with the cyclic occurrence of depression, anxiety, and emotional lability that occurs 1 week prior to menses and dissipates within 1 week after the completion of menses. These symptoms are most pronounced in the luteal phase of the menstrual cycle and may also include anger, irritability, and internal tension. Physical symptoms include breast tenderness, abdominal bloating, and headaches. To meet the DSM criteria, the patient must have ≥5 of the above signs or symptoms. Approximately 3-8% of menstruating women meet the criteria for this condition. SSRIs are the medication class of choice for PMDD. Alternative options include calcium supplementation, pyridoxine, magnesium, and vitamin E.
A 67-year-old man with emphysema comes to his healthcare provider for a routine checkup. He reports that his symptoms have remained stable, with frequent coughing and dyspnea, despite compliance to his medications, which include oral prednisone, aminophylline, inhaled beta 2 agonists, and rotating courses of antibiotics. On physical examination he has a barrel-chest and distant breath sounds in both lung fields with soft expiratory wheezes. An arterial blood gas reveals an arterial pH of 7.32, a pCO2 of 47 mm Hg, and a pO2 of 53 mm Hg. Which of the following would most likely prolong his survival? A. Continuous home oxygen therapy B. High-dose oral prednisone C. Influenza vaccine D. Pneumococcal vaccinE E. Tracheostomy
The correct answer is A. This patient has resting hypoxia and therefore would benefit in terms of survival with the use of continuous, low-flow home oxygen therapy. Care must be taken as to not deliver high concentrations of oxygen, as that would suppress his respiratory drive. Oxygen saturation in these patients should be maintained just above 90%. Continuous home oxygen therapy in this patient will not only improve symptoms, but also prolong survival. Indications for home oxygen therapy are as follow: Arterial pO2 (PaO2) less than or equal to 55 mmHg, or an arterial oxygen saturation (SaO2) less than or equal to 88 percent. PaO2 less than or equal to 59 mmHg, or an SaO2 less than or equal to 89 percent, if there is evidence of cor pulmonale, right heart failure, or erythrocytosis (hematocrit >55 percent). High-dose oral prednisone (choice B) may improve symptoms in the short-term during an acute exacerbation, but it has no role in the long-term management of patients with chronic obstructive pulmonary disease (COPD) and it does not prolong survival. Choices C and D are appropriate measures to take in every COPD patient to prevent pneumococcal and influenza pneumonias respectively, but they do not provide the same prolongation of survival. A tracheostomy (choice E) plays no role in the management of COPD, unless the patient is in an intensive care unit setting and cannot be weaned from a respirator. PEARL: COPD patients with severe disease tend to have chronic hypoxemia and therefore long-term home oxygen therapy should be initiated as it improves survival and quality of life in these patients. The only other intervention that would decrease mortality is smoking cessation. Remember the only 2 interventions that will improve survival in patients with COPD are: home oxygen therapy and smoking cessation .
A 27-year-old woman is admitted to the hospital because of 4 days of joint pain. Initially she had diffuse joint pain that would "travel" from her wrists to her elbows and to her knees. Over the last 24 hours, however, she has had intense pain and swelling localized to her right knee. She denies any trauma to the area. She denies any fevers or chills but admits to mild dysuria that she attributed to a urinary tract infection. Further history reveals that she has had unprotected sex with 4 partners in the last 3 months. Her temperature is 37.0°C (98.6°F), blood pressure 120/70 mm Hg, pulse 113/min, and respirations 18/min. The right knee is swollen and tender. She has scattered necrotic pustules on her palms and soles. Arthrocentesis is performed and a complete blood count and blood cultures are obtained. Hemoglobin:14.0 mg/dLLeukocytes:10,000/mm3Platelets:170,000/mm3Hematocrit:36% Gram stain of the synovial fluid is shown. Which of the following is the most appropriate next step in management? A. Begin therapy with IV ceftriaxone (Rocephin) B. Begin therapy with IV nafcillin C. Begin therapy with IV penicillin D. Begin therapy with IV vancomycin E. Hold antibiotics and wait for culture and sensitivities results
The correct answer is A. This patient's presentation is classic for disseminated gonococcal disease. Disseminate gonococcal disease begins as a migratory arthritis and evolves into a purulent monoarthritis. Fever, elevated WBCs, and genitourinary symptoms are variable. Necrotic papules on the extremities can be a valuable clue to the diagnosis. The patient's history of unprotected sex and dysuria should provide further clues to diagnosis. NAAT (nucleic acid amplification test) is recommended on the first urine in the morning in order to make this diagnosis. Health care providers should culture the rectum, cervix, urethra, and pharynx as well as obtain blood cultures if a drug resistant strain of gonorrhea is suspected. Initial treatment with ceftriaxone is now recommended as first-line treatment. Typically, this is accompanied with a single one gram of azithromycin. Parenteral antibiotics are typically given for 7 days. . It would also be prudent to cover for the possible coexisting Chlamydia infection with the single dose of azithromycin. Nafcillin (choice B) is the first-line agent for staphylococcal septic arthritis. Our patient's history strongly suggests gonococcal arthritis and the synovial fluid Gram stain further supports the diagnosis. Gonococcal penicillin-resistance is becoming increasingly prevalent, so penicillin is not an appropriate initial antibiotic (choice C) . Historically, penicillin was first-line treatment, but this is no longer the case. Vancomycin (choice D) should be reserved for patients who are at risk for methicillin-resistant Staphylococcus aureus, such as IV drug users, nursing home patients, or in-patients who acquired the infection in the hospital. Vancomycin is not utilized in the treatment of gonococcal disease. Withholding antibiotics (choice E) is incorrect. Septic arthritis can be very destructive to the involved joint. Patients who have septic joints must be admitted to the hospital and treated aggressively and quickly with antibiotics. Waiting for culture results can be dangerous and could result in permanent joint damage. PEARL: Gonorrhea can lead to systemic symptoms such as polyarthritis, which indicates disseminated disease. The joints most commonly involved include the wrists, ankles, and small joints of the hands and feet. Signs of disseminated disease can also include conjunctivitis, fever, and skin lesions at the extremities. Treatment includes ceftriaxone (Rocephin) plus a single one-gram dose of azithromycin Disseminated gonococcal infection (DGI) is the most common cause of acute nontraumatic monoarthritis or oligoarthritis in young adults. Patients who have DGI typically present with one of 2 syndromes: A triad of tenosynovitis, vesiculopustular skin lesions, and polyarthralgias without purulent arthritis Purulent arthritis without associated skin lesions Gonococcal arthritis is a common cause of septic arthritis in which the organism cannot be cultured on routine culture media. Cultures of synovial fluid tend to be positive in <50% of cases of gonococcal arthritis. Because of this, a negative synovial culture does not rule out gonococcal arthritis. Affected adults are sexually active individuals and typically present with fever, chills, skin lesions, polyarthralgias, and tenosynovitis evolving into a persistent monoarthritis or oligoarthritis. The joint aspirate should be cultured for N. gonorrhoeae when the history is suggestive. NAAT of the first morning voided urine can prove that gonorrhea is present. The yield can be increased if plates of chocolate agar or Thayer-Martin medium are inoculated with synovial fluid at the bedside along with cultures from clinically appropriate sites (e.g., the pharynx, urethra, cervix, rectum, and skin lesions). Blood cultures are often positive in patients who have DGI presenting with tenosynovitis and skin lesions alone, but are frequently negative if a joint effusion is present. The infected fluid is usually purulent with an average leukocyte count (most of which are neutrophils) of 50,000ndash150,000 cells/mm3. The likelihood of septic arthritis increases with rising synovial fluid leukocyte count. Nucleic acid hybridization tests and nucleic acid amplification tests (NAAT) are now generally recommended as the first-line method of testing. Urine, urethral, cervical, and vaginal specimens can be utilized.
A 74-year-old woman, who has been followed for the past 25 years for chronic obstructive pulmonary disease (COPD) comes to the emergency department complaining of 48 hours of temperatures of 38.6° C (101.4° F) and worsening shortness of breath. She has a chronic productive cough, which has become more copious. On physical examination she has rhonchi and increased fremitus in the posterior mid-lung field. Patient has a long-standing history or smoking and continues to smoke. A Gram's stain reveals many epithelial cells and multiple gram-positive and gram-negative organisms; no neutrophils are seen. Which of the following is the most likely organism causing the symptoms? A. Escherichia coli B. Haemophilus influenzae C. Klebsiella pneumoniae D. Mycobacterium tuberculosis E. Mycoplasma pneumoniae
The correct answer is B. This patient, with a long history of chronic obstructive pulmonary disease (COPD), has evidence of a community-acquired pneumonia. The common organisms causing pneumonias in patients with COPD are Streptococcus pneumoniae, Haemophilus influenzae, and Moraxella catarrhalis. Patients who smoke are often colonized with Haemophilis influenza . This patient has no other history suggestive of Escherichia coli (choice A) infection elsewhere (such as in the urinary tract), and primary E. coli pneumonia is rare. Klebsiella pneumoniae (choice C) is typically found in alcoholic patients and the pneumonia often causes cavitations in the lung. There is no evidence of tuberculosis (choice D) by history. Tuberculosis usually presents with a more chronic presentation. Furthermore, it would generally be found as an upper lobe infiltrate, consistent with reactivation tuberculosis. Much less commonly, tuberculosis may present as a primary infection, but this is generally seen in patients with an underlying immunocompromised state. In the setting of primary tuberculosis, a lower lung field pneumonia is in fact possible. Mycoplasma pneumoniae (choice E) does not present with a lobar consolidation and is generally a disease of younger people who present with fever, malaise of at least several days duration, and a nonproductive cough. The chest x-ray film in a patient with Mycoplasma pneumonia would classically reveal faint bilateral interstitial infiltrates. PEARL: Strep. pneumoniae is the most common cause of CAP. Non-typeable H. influenzae is an important cause of pneumonia in elderly adults and in patients with underlying pulmonary disease, such as cystic fibrosis and COPD and in those who continue to smoke. Mycoplasma pneumoniae is one of the most common causes of atypical pneumonia and is commonly seen in school-aged children, military recruits, and college students. Moraxella is a gram-negative diplococcus that can cause lower respiratory tract infections in immunocompromised persons and in patients with COPD.
A 65-year-old woman is admitted to the hospital for constant, severe abdominal pain that has worsened over the previous week. She has no other associated symptoms, such as nausea or vomiting, but has noticed that her daily urine output has sharply decreased. She has had a constant desire to urinate, but when she tries, only a small amount of bloody urine is discharged. The patient is a long-time smoker, having smoked 3 packs per day for 45+ years, though she claims to have quit 2 days ago. Bladder ultrasound in the emergency department reveals a large mass consistent with bladder cancer, as well as significant urinary retention. Which of the following is most likely to be detected upon imaging the patient's genitourinary system? A. Bilateral hydronephrosis B. Bladder dilation C. Bladder dyskinesis D. Unilateral hydronephrosis E. Ureteral dilation
The correct answer is A. Urinary retention is most often caused by an anatomic obstruction to urine outflow. In men, this is often caused by benign prostatic hyperplasia (BPH). Women have a variety of causes. The common manifestation of prolonged urinary retention is bilateral hydronephrosis caused by urinary retention and pressure increases in the urinary system. A large bladder mass would compress both distal ureters leading to inability of the kidneys to empty with resultant bilateral hydronephrosis. The bladder is a very muscular organ. Increases in pressure do not cause bladder dilation (choice B) , but rather hypertrophy. Bladder dyskinesis (choice C) , like ventricular dyskinesis, would most likely be seen in an area of focal bladder injury. This most often results from external impingement on the bladder wall. Unilateral hydronephrosis (choice D) is most often encountered in cases of ureteral obstruction in which only one kidney suffers from the increased pressure. Ureteral dilation (choice E) would not be seen in urinary retention until very late in the course. It is an uncommon finding because most patients present to a health care provider prior to this late stage. PEARL: Bladder cancer generally presents in patients age >60. Think of bladder cancer in patients who have a history of smoking, urinary obstruction (which can lead to hydronephrosis), and/or hematuria. Bladder cancer is the fourth most common cancer in men and the ninth most common cancer in women. 90% of bladder cancer is transitional cell carcinoma.
A 15-year-old girl comes to the clinic because of mild vulvovaginal itching and a foul-smelling vaginal discharge. She is otherwise generally healthy and takes no medications. She is sexually active with one male partner and says that they use condoms sporadically. Her sexual partner is asymptomatic. Examination shows deeply erythematous vaginal mucosa and a diffusely inflamed and friable cervix. There is a moderate amount of gray-green vaginal discharge. A microscopic image of the vaginal discharge is shown A. Metronidazole (Flagyl), 2 g orally, single dose B. Clindamycin (Cleocin) vaginal cream, 1 applicator full in vagina at night for 3 nights C. Benzathine penicillin G, 2.4 million units IM in a single dose D. Azithromycin (Zithromax), 1 g orally in a single dose and doxycycline 100 mg orally BID for 7 days E. Clotrimazole (Gyne-Lotrimin), 1 applicator full in the vagina at night for 3 nights
The correct answer is A.This adolescent has trichomoniasis, which is caused by Trichomonas vaginalis. It is typically sexually transmitted. Metronidazole is the treatment of choice. This patient should avoid sexual intercourse until both she and her partner have been treated and are asymptomatic.. Clindamycin (Cleocin) (choice B) is one of the accepted treatments for bacterial vaginosis. It will not eradicate trichomoniasis. Benzathine penicillin (choice C) is the drug of choice for the treatment of syphilis. Zithromax and doxycycline (choice D) are the drugs of choice to treat chlamydial infections and gonococcal infections, not trichomoniasis. Clotrimazole (Gyne-Lotrimin) (choice E) is one of the accepted treatments for candidiasis, but this patient has trichomoniasis. PEARL: Trichomoniasis classically presents with profuse, yellow-green vaginal discharge that tends to be frothy, occasionally with vaginal and vulvar pruritis. On visual inspection there is vaginal erythema and mottling of the cervix, classically described as a "strawberry cervix"; the male partner is usually asymptomatic.
A 36-year-old man develops rapid mental status deterioration 2 days after sustaining a femoral fracture in a skiing accident. Physical examination shows multiple petechiae in the anterior chest and abdomen. On the third day, the patient lapses into coma and dies. Postmortem examination of the brain reveals numerous petechial hemorrhages in the corpus callosum and centrum semiovale. Which of the following is the most likely diagnosis? ANSWER SELECTION Diffuse axonal injury Fat embolism Septic embolism Systemic thromboembolism Watershed infarction
The correct answer is B. The clinical manifestations in the scenario are consistent with fat embolism. This complication is frequently associated with fractures of long bones, but this is usually asymptomatic. Fat embolism mainly affects the lungs and the brain, and the clinical picture consists of dyspnea, tachycardia, and mental status changes. Only rarely does this condition lead to death. In the lungs, fat emboli can be visualized histologically. In the brain, multifocal petechiae in the white matter represent the most common pathologic change. Diffuse axonal injury (choice A) is one of the most common complications from traumatic brain injury. It involves the central white matter, especially the corpus callosum and cerebral peduncles. It is sometimes associated with small petechiae in these areas. The patient may develop coma a few hours to days after head trauma. Septic embolism (choice C) results from septic emboli lodging in the terminal intraparenchymal arteries of the brain. It leads to multiple cortical infarcts, usually of the hemorrhagic type. The white matter is spared. Systemic thromboembolism (choice D) is usually of cardiac origin, for example, in patients who have cardiac arrhythmias with thrombi in the right atrium or ventricle. Thromboemboli in the brain cause ischemia/ infarction in the cortex. Watershed infarction (choice E) is often seen in patients suffering from acute hypotensive episodes, especially if the circle of Willis is already compromised by atherosclerotic change. The cortical regions at the border zone between different vascular territories (e.g., between the distribution of the anterior and middle cerebral arteries) undergo ischemic necrosis. PEARL: Fat embolism is a clinical diagnosis. It usually presents with a history of trauma to long bones or pelvis (including orthopedic procedures). Symptomatic patients present with respiratory distress, neurologic impairment, and a petechial rash over the upper body. Petechial rash, present in only 20% of cases, is considered pathognomonic in the setting of long-bone trauma.
A 19-year-old woman with no significant past medical history comes in for a pre-college physical examination required by her school. She has no complaints, and her review of systems is negative. Her temperature is 37.0°C (98.6°F), blood pressure 120/80 mm Hg, pulse 80/min, and respirations 10/min. Physical examination reveals a single nodule on the left lobe of the thyroid gland. Her thyroid-stimulating hormone is within normal limits. Which of the following is the most appropriate next step in management? A. CT scan of the thyroid gland B. Fine-needle biopsy C. Repeat examination in 6 months D. Thyroid uptake scanning
The correct answer is B. The first test for a thyroid nodule is the thyroid-stimulating hormone test; if normal, the most appropriate next step is fine-needle aspiration which can be done safely and without complication. An ultrasound can be used to differentiate between solid and cystic structure and can be used to guide the needle aspiration. The fine-needle biopsy provides a definitive diagnosis in most cases and is the most helpful test in guiding management. CT scanning of the thyroid gland (choice A) is important in the workup of a patient who has a goiter and is complaining of obstructive symptoms such as shortness of breath or difficulty swallowing. It has no role in the initial workup of a thyroid nodule. If the patient has thyroid cancer, the CT scan of the thyroid can be used to help stage the cancer. Serial physical examinations (choice C) are not acceptable management. Thyroid cancer is curable in its early stages, so efforts should be made to rule out carcinoma in these patients. Thyroid uptake scanning (choice D) may be useful; however, a fine-needle aspiration is generally considered the next best step in this case. Approximately 20% of nonfunctioning thyroid nodules (cold appearance) prove to be malignant; functioning nodules (hot nodules) are seldom malignant. The thyroid uptake scanning provides a percentage number (high or low) that demonstrates the activity of the gland and is used to differentiate between Graves disease (high uptake), multinodular goiter (patchy uptake) and subacute thyroiditis (low uptake). PEARL: A patient who has a thyroid nodule should have a full thyroid function evaluation performed. The first test performed should be the TSH; a suppressed level suggests hyperthyroidism and a hot nodule, which lessens the possibility for cancer. The next test should be a fine-needle biopsy (if the TSH is not suppressed). Histology will identify whether the patient has cancer and also will identify the type of cancer that the patient has. A thyroid scan can help to identify the nodule and whether it is hyper- or hypofunctioning.
PANCE® PANCE® Questionof the Day forOctober 10 QUESTION A 32-year-old man has had asthma for the past 9 years. The symptoms are frequently exacerbated by changes in the weather and household allergens. He has had 2 emergency department visits over the past year, where he was started on oral corticosteroids, but he has not required hospitalization. He uses an albuterol inhaler about 4 times per week for symptom relief and has symptoms at night about 3 times a month. Which of the following would be the most appropriate therapy to maintain remission between his asthmatic attacks? A. Aminophylline B. Beclomethasone inhaler C. Cromolyn nasal spray D. Metaproterenol inhaler E. Oral prednisone
The correct answer is B. This patient has mild persistent asthma. Low dose Inhaled steroids should be the mainstay of therapy to maintain remission between asthmatic episodes in a patient with mild persistent asthma. They effectively reduce airway inflammation, whereas the beta-2 agonist inhalers are more effective for short-term relief of symptoms. The most likely side effect you would find on the boards from steroid inhalers is the development of oral candidiasis, which can be prevented by prompt mouth rinsing after inhaler use. Aminophylline (choice A) has no role in the maintenance therapy of asthma and has very limited use during an acute attack as well. Cromolyn (choice C) is not effective in maintaining remissions. However, it is useful as a prophylactic measure before initiating exercise in those patients whose trigger for bronchoconstriction is exercise. A metaproterenol inhaler (choice D) is of value in reducing symptoms during an acute attack but is not effective in preventing relapses. Oral prednisone (choice E) is effective in preventing relapses but is reserved for patients who have had severe refractory asthmatic attacks and should not be used for patients with a milder course. PEARL: The preferred controller medication in patients with mild persistent asthma is a low dose inhaled steroid which can reduce the frequency symptoms and the need for short-acting beta agonists. Low dose inhaled steroids also improve the quality of life and reduce the risk of serious exacerbations. Low dose inhaled steroids do NOT prevent progressive loss of lung function. Mild Intermittent Asthma is characterized by the following: Daytime asthma symptoms occurring 2 or fewer days per week 2 or fewer nocturnal symptoms per month Use of short-acting beta agonists fewer than twice weekly No interference with normal activities between exacerbations FEV1 measurements between exacerbations that are consistently within the normal range (ie, ≥80 percent of predicted normal) FEV1/FVC ratio between exacerbations that is normal (based on age-adjusted values) One or less exacerbations per year requiring oral glucocorticoids Mild Persistent Asthma is characterized by the following: Symptoms more than twice weekly (although less than daily) 3 to 4 nocturnal symptoms per month Use of short-acting beta agonists to relieve symptoms more than twice weekly (but not daily) Minor interference with normal activities FEV1 measurements within normal range (≥80 percent of predicted normal) FEV1/FVC ratio is normal (based on age-adjusted values) 2 or more exacerbations per year requiring oral glucocorticoids Moderate persistent Asthma is characterized by the following: Daily symptoms of asthma Nocturnal symptoms more than once per week Daily use of short-acting beta agonists for symptom relief Some limitation in normal activity FEV1 between 60 and 80 percent of predicted FEV1/FVC reduced below normal (based on age-adjusted values) 2 or more exacerbations per year requiring oral glucocorticoids Severe Persistent Asthma is characterized by one or more of the following: Symptoms of asthma throughout the day Nocturnal symptoms nightly Use of short-acting beta agonists for symptom relief several times per day Extreme limitation in normal activity FEV1 <60 percent of predicted FEV1/FVC reduced below normal (based on age-adjusted values) 2 or more exacerbations per year requiring oral glucocorticoids
Six days after an uncomplicated left nephrectomy, a 42-year-old man complains of moderate abdominal pain and diarrhea. He is stooling approximately 6 times a day with profuse watery diarrhea. The patient also experiences painful cramps during these episodes. In other medical history, the patient has a history of hypertension and elevated cholesterol. His medications include metoprolol (Lopressor) and simvastatin (Zocor). He denies any recent cardiopulmonary history and any history of chronic gastrointestinal disorders. His chart indicates that he received a 2-day regimen of cefazolin (Ancef) for perioperative skin prophylaxis. His temperature is 38.3°C (101.0°F). Stool is positive for fecal leukocytes and occult blood. The diagnosis is established with additional studies and he is given treatment. Unfortunately he fails to respond to therapy and requires surgery. An image of a segment of his colon is shown. Which of the following is the most likely pathogen? A. Bacillus cereus B. Clostridium difficile C. Escherichia coli 0157:H7 D. Shigella E. Yersinia enterocolitica
The correct answer is B. This patient is experiencing C. difficile colitis, also known as pseudomembranous colitis. This type of diarrhea is common in hospitalized patients; at least 30% of hospitalized patients are colonized with the organism. Certain antibiotics have a predilection for causing C. difficile colitis: clindamycin, ampicillin, amoxicillin, and cephalosporins have a high incidence of causing this infection but it can be linked with any of the antibiotics. These antibiotics suppress normal gut flora, allowing Clostridium to proliferate. Two endotoxins (A and B) cause the symptoms and the colonic pathology noted in the vignette. Diagnosis is made by identifying stool containing C. difficile toxin. ; Sigmoidoscopy also provides additional diagnostic information, particularly the presence of plaques (also called pseudomembranes). Treatment is oral metronidazole or vancomycin. Surgery is sometimes indicated for recurrences that are unresponsive to therapy or when complications such as toxic megacolon occur. B. cereus (choice A) is a spore-forming gram-negative rod that causes diarrhea similar to that seen with C. difficile colitis. It is usually associated with food poisoning, however, particularly on reheated grain foodstuffs (such as fried rice). Treatment is symptomatic. Patients who have this pathogen develop nausea and vomiting within 6 hour of ingestion of this pathogen, and this is followed by diarrhea. There is no fever associated with this. E. coli 0157:H7 (choice C) causes severe abdominal pain and severe bloody diarrhea. It is usually obtained by consumption of raw beef or unprocessed milk. This organism also has been implicated in hemolytic-uremic syndrome, a potentially deadly syndrome involving hemolysis and kidney failure. For hemolytic uremic syndrome to develop, the patient typically must be treated with antibiotics after ingestion of this pathogen. Shiga-like toxins cause direct damage to renal glomerular endothelial cells, which lead to the kidney damage. Treatment usually involves parenteral antibiotics. Shigella (choice D) species cause fever, cramps, and diarrhea that can be watery but later develop into bloody, mucus-containing stools. It is usually obtained by poor hygiene, infected food, and transmission by flies. Children attending daycare are the most likely to acquire this condition. Treatment is symptomatic if mild, oral fluoroquinolones or trimethoprim-sulfamethoxazole if more severe. The symptoms of Y. enterocolitica (choice E) infection are almost identical to those of Salmonella. Yersinia is typically acquired by ingestion of contaminated pork, beef, milk, or cheese. If it is associated with abdominal pain, the pain is classically located in the right lower quadrant, as this pathogen has a predilection for the cecum and terminal ileum. It is also known to cause mesenteric adenitis. Treatment is symptomatic. This condition is sometimes mistaken for appendicitis.. PEARL: The 2 main risk factors for Clostridium difficile colitis are recent antibiotic use (which disrupts the normal bowel flora and allows this native pathogen to replicate unchecked) and recent hospitalization (which increases risk for acquiring this pathogen, primarily because of poor handwashing techniques). Patients who have this pathogen typically present with diarrhea, abdominal pain, and leukocytosis. There may also be associated fever, abdominal tenderness, and abdominal distention. Identification of the stool cytotoxin tissue culture assay is diagnostic. Identification is usually done by stool enzyme immunoassay for toxins A and B. Patients may also have + fecal occult blood testing on the stool, but this is not specific for the diagnosis. Diagnosis can also be made via sigmoidoscopy or colonoscopy because of its classic appearance with pseudomembranes. Treatment involves discontinuing the antibiotic and administering oral metronidazole (Flagyl) or vancomycin. Clostridium difficile can survive on the skin when using antibacterial gel for hand hygiene so the recommendation is to use hand washing with soap and water to prevent transmission to others.
PANCE® PANCE® Questionof the Day forOctober 14 QUESTION A 70-year-old man with an 88 pack-year smoking history and hypertension has slowly progressive shortness of breath over the last 1 to 2 years. Physical examination reveals a cachectic man who is in mild respiratory distress using accessory muscles and ''pursed lip'' breathing. He is not cyanotic. His breath sounds are decreased without wheezing or rhonchi. His heart sounds are distant. He has no edema. A chest x-ray shows hyperinflation of lung fields with a small heart, flattened diaphragm, and no infiltrates or edema. Pulmonary function testing is most likely to reveal which of the following findings? A. Bronchodilator response of approximately 15 to 20% B. Decreased diffusion capacity of the lung for carbon monoxide C. Decreased residual volume D. Decreased total lung capacity E. Increased FEV1/FVC ratio
The correct answer is B. This patient most likely has COPD, which includes emphysema and chronic bronchitis. Although many patients have features of emphysema and chronic bronchitis, they have separate features. This patient is described as the classic "pink-puffer" with emphysema. Patients with emphysema-type changes in the lung have irreversible permanent dilation of distal airspaces with cartilage destruction. Low diffusion capacity of the lung for carbon monoxide (DLCO) is expected in emphysema, whereas patients with chronic bronchitis have a normal DLCO. Emphysema leads to the destruction of vasculature in the alveolar septa which leads to a decreased diffusion of CO into the blood. This leads to an increased CO in the exhaled gas. A bronchodilator response (choice A) is the finding of asthma. Chronic obstructive pulmonary disease is characterized by irreversible airway obstruction. Patients with emphysema have air-trapping secondary to decreased elastic recoil in their lungs. This will show increased residual volume (choice C) and total lung capacity (choice D) . The ratio of forced expiratory volume in 1 second (FEV1) to forced vital capacity (FVC) is used to measure air movement in and out of the lungs. If this ratio is 80% of expected or less, a diagnosis of obstructive lung disease is suspected. Because our patient has an obstructive lung disease, we would expect FEV1/FVC to decrease, not increase (choice E) . PEARL: Patients who present to a healthcare provider with a combination of chronic productive cough, dyspnea, and/or known inhalational exposure (tobacco, dust, chemicals, etc.) should be referred for pulmonary function testing (PFTs) for evaluation of possible COPD. PFTs will reveal an airflow obstruction with a FEV1/FVC ratio of less than 0.70 in COPD. The diffusing capacity of carbon monoxide (DLCO) is a nonsensitive and nonspecific test and therefore cannot be used to identify patients with mild COPD, but would be decreased in proportion to the severity of the emphysema in more severe cases. PEARL: Learn how to recognize the typical presentation for emphysema: flattened diaphragm, small heart, hyperinflated lungs, and pursued-lips breathing. Patients with emphysema-type changes in the lung have irreversible permanent dilation and destruction of distal air spaces, which will explain a low diffusion capacity of the lung for carbon monoxide (DLCO). Patients with chronic bronchitis have a normal DLCO.
A 2-year-old girl is brought to the emergency department because she looked "unwell" and has not wanted to get out of bed for 24 hours. She had a diarrheal illness 1 week ago marked by fever and bloody diarrhea, from which she had recovered. Two days ago, however, she developed reduced appetite and lethargy. She has had only 2 wet diapers in the last 36 hours. She was previously a well child and at the regular clinic visits normal growth and development were noted. Examination shows an irritable, pale child who has petechiae and edema. Vital signs reveal the blood pressure 102/74 mm Hg, pulse 136/min, and respirations 26/min. Lung and abdominal examinations are normal. Laboratory studies show: Hemoglobin7.1g/dLWBCs222 x 109/L (normal 5,000-10,000)Differential:Polymorphs38%Lymphocytes52%Monocytes8%Platelets80 x 109/LBlood filmFragmented red blood cellsProthrombin time12.1 secPartial thromboplastin time32 secSerum sodium132 mEq/LSerum potassium6.3 mEq/LSerum chloride106 mEq/LSerum bicarbonate16 mEq/LBlood urea nitrogen32 mg/dLSerum creatinine3.1 mg/dL Which of the following is the appropriate next step in management? A. Administer a normal saline bolus of 20 gmL/k B. Administer methylprednisolone C. Begin dialysis D. Begin plasmapheresis E. Transfuse red blood cells
The correct answer is C. This patient has presented in acute renal failure, which is manifested by oliguria (<400-500 mL/24h) elevated blood urea nitrogen and creatinine, and hyperkalemia. Given the clinical picture, the most likely etiology is hemolytic uremic syndrome (HUS). HUS is the most common cause of acute renal failure in young children and most frequently follows an episode of gastroenteritis caused by an enterohemorrhagic strain of Escherichia coli (0157:H7). Endothelial injury to the kidneys leads to renal failure, whereas damage to red blood cells and platelets in the kidneys leads to the thrombocytopenia and microangiopathic hemolytic anemia. The management is supportive care of the renal failure, which is self-limited in most cases. Given the marked elevation of creatinine as well as hyperkalemia in this patient, the most appropriate choice would be to initiate dialysis. Children who present with oliguria may benefit from a normal saline bolus (choice A) if the intravascular volume is depleted. The patient in this vignette demonstrates signs of fluid overload, however, and a bolus of IV fluids is inappropriate in this setting. Steroids such as methylprednisolone (choice B) are used for renal diseases such as lupus nephritis and nephrotic syndrome. They have no role in the treatment of HUS Plasmapheresis (choice D) is useful in adult patients who have thrombotic thrombocytopenic purpura, but not in children who have HUS. Because of the presence of fluid overload and hyperkalemia, red blood cell transfusion (choice E) is not safe in this patient until peritoneal dialysis has been instituted. PEARL: Hemolytic uremic syndrome (HUS) is common after infection with Escherichia coli (O157:H7). Endothelial injury to the kidneys leads to renal failure, which is diagnosed by oliguria, elevated blood urea nitrogen and creatinine, and hyperkalemia. In the instance of marked elevation of potassium and creatinine, the first step would be dialysis, then continued supportive treatment.
A 3-year-old boy is brought to the office by his parents for a regular health supervision examination. His condition has not changed significantly since his last visit 6 months earlier. He still prefers to play by himself and rocks back and forth constantly. The mother has finally adjusted to the fact that her son shows no signs of emotional attachment to her. On physical examination the boy is noncooperative and utters unintelligible mumbles when the healthcare provider approaches him. The healthcare provider fails to establish any eye contact with him. Suddenly the boy starts screaming and runs over to the wall and repetitively bangs his forehead against it. Which of the following is the most likely diagnosis? A. Asperger disorder B. Attention deficit hyperactivity disorder C. Autism D. Mental retardation E. Rett syndrome
The correct answer is C.This is a typical clinical presentation of autism, a developmental disorder characterized by impaired social relatedness, deficits in verbal and nonverbal communication, and unusual responses to the environment. It develops before age 30 months and is more common in boys (4:1). The cause is unknown. Clinical features include failure to attach as an infant, delayed or absent social smile, and failure to anticipate interaction with the caretaker. Patients demonstrate a delay in verbal and nonverbal communication skills. Behavioral characteristics can also include obsessional or perfectionistic behavior. Stereotypical movements and a need for sameness and routine are characteristic. Outbursts of anger are common, as well as self-injurious behavior. Autistic children are content to play alone. They may or may not be mentally retarded. Treatment is behavioral and educational, and is tailored to meet individual needs. Pharmacotherapy may be used to treat some target behaviors. Prognosis is poor and a very small number of these children will grow up to be marginally self-sufficient. The vast majority end up institutionalized. A better prognosis is associated with patients who demonstrate functional speech and higher intelligence. Asperger disorder (choice A) is a pervasive developmental disorder that differs from autism in that the patients are more communicative, appear more socially aware, and do not have language impairments found in autism. They do have social impairments, repetitive behaviors, and sometimes obsessional interests. Early language development is not delayed in this condition. These children will grow up to have average or above average IQs. Attention deficit hyperactivity disorder (choice B) is a disruptive behavior disorder. Diagnosis requires either serious inattention or hyperactivity and impulsivity severe enough to cause significant distress or functional impairment. Symptoms must begin before the age of 7 and these symptoms must occur in more than one environmental setting. The single most effective treatment is stimulants. Mental retardation (choice D) affects 13% of the population and occurs more commonly in boys. The patient must have both impaired functioning and IQ <70, with the onset age <18. Most cases have no obvious cause. Rett syndrome (choice E) is an X-linked dominant disorder affecting only girls. Development is normal until age 1, when language and motor milestones regress and an acquired microcephaly is seen. Hand wringing and sighing are characteristic, and patients develop autistic behavior. PEARL: Autism is a childhood developmental disorder in which children have social communication difficulties and show repetitive, obsessional, or perfectionistic behavior. Speech delay is present. Boys are affected much more commonly than girls. Behavioral patterns include restricted, repetitive, and stereotypical behaviors. There is an increase in the occurrence of seizures and mental retardation in these children.
PANCE® PANCE® Questionof the Day forOctober 12 QUESTION A 79-year-old man weighing 75 kg with emphysema is intubated in the intensive care unit because of respiratory failure after developing adult respiratory distress syndrome secondary to an Escherichia coli bacteremia. This event followed an untreated urinary tract infection. The ventilator is set to a respiratory rate 20/min, tidal volume 450 mL/breath, and pO2100%. If these settings are continued for long term use, the patient has increased risk for developing which of the following complications? A. Congestive heart failure B. Jugular venous distention C. Pulmonary embolus D. Pulmonary fibrosis E. Tension pneumothorax
The correct answer is D. High concentrations of inspired oxygen delivered through a ventilator may lead to pulmonary fibrosis, which becomes irreversible. In the setting of adult respiratory distress syndrome (ARDS), if the inspired fraction of oxygen cannot be lowered without producing hypoxia, the addition of positive-end expiratory pressure (PEEP) is indicated. Although PEEP does increase the risk of both barotrauma and hypotension by impairing right-sided heart filling, it is indicated to prevent the development of oxygen toxicity, which may result in irreversible pulmonary fibrosis. Congestive heart failure (choice A) can occur as a complication of the patient's longstanding underlying pulmonary disease, but the incidence would not likely be increased because of his ventilator settings. Jugular venous distension (choice B) can be a marker for either right heart congestive failure (see choice A discussion) or tension pneumothorax (see choice E discussion), but would not be a likely complication of a high pO2. Pulmonary embolus (choice C) would more likely be related to prolonged bed rest with resultant venous thrombosis. Tension pneumothorax (choice E) would be more likely to occur if the tidal volume were significantly greater than 450 mL/breath (corresponding to the optimal flow rate of 6 mL/ kg). PEARL: High concentration of oxygen can be toxic to lung tissue and therefore the lowest possible fraction of inspired oxygen (FiO2) should be used to maintain adequate oxygenation. In high concentrations, oxygen can lead to absorptive atelectasis, accentuation hypercapnia, airway injury, and lung parenchymal injury. The oxygen toxicity is caused by inflammation which leads to cell death. It should be noted that there is NO threshold FiO2 or duration of oxygen supplementation below which oxygen toxicity cannot occur. Titration of FiO2 should be performed to maintain adequate oxygenation. A goal is to aim for a PaO2 greater than 60 mmHg and an oxygen saturation above 90%, but these recommendations are not as steadfast as patients with coronary artery disease or pulmonary hypertension will not tolerate hypoxemia very well. In most situations, physiologic PEEP is indicated for intubated patients to minimize risk of end-expiratory alveolar collapse and ventilator-associated pneumonia. High concentrations of inspired oxygen delivered through a ventilator may lead to irreversible pulmonary fibrosis. In the setting of adult respiratory distress syndrome (ARDS), PEEP (positive end-expiratory pressure) is indicated to prevent the development of oxygen toxicity.
An 8-year-old boy has left leg pain and a high fever, and has been refusing to walk since yesterday. On physical examination, his temperature is 39.8ordmC (103.6ordmF), blood pressure 122/68 mm Hg, pulse 102/min, and respirations 20/min. The left femur is tender to palpation 3 cm above the left knee and there is marked soft-tissue swelling. Radiograph of the left leg is normal. Bone scan shows increased uptake around the metaphysis of the left femur. Which of the following is the most likely pathogen? Escherichia coli Haemophilus influenzae Salmonella Staphylococcus aureus Streptococcus pneumoniae
The correct answer is D. Osteomyelitis is a pyogenic infection of the bone. Pathogenesis is similar to septic arthritis, with the origin of infection occurring from hematogenous spread, direct extension of a local infection, or direct inoculation of bone either from trauma or surgical manipulation. In children the most frequent presentation is acute hematogenous spread. The most common location of osteomyelitis is the metaphysis of the distal femur and proximal tibia. The most prevalent pathogens are the same as those seen in septic arthritis. Staphylococcus aureus is the most common pathogen followed by group A beta-hemolytic streptococci. Neonates are at risk for group B beta-hemolytic streptococci, but risk is dramatically lower since pregnant women are routinely tested and treated for this pathogen prior to labor. Haemophilus influenzae may occur in infants and young children, but it is not seen as frequently as in septic arthritis. In addition, children who have H. influenzae osteomyelitis usually have fever and concomitant joint infection. Patients who have puncture wounds of the foot are susceptible to Pseudomonas aeruginosa osteomyelitis. Patients with sickle cell disease are at risk for infection by Salmonella and other gram-negative bacteria. Most patients who have osteomyelitis will present with a chief complaint of fever and bone pain. The pain is usually severe, constant, and aggravated by motion. The older the child, the more exquisite the point tenderness. That is because the bone has a thicker metaphyseal cortex with a dense fibrous periosteum. Localized swelling, warmth, and erythema are signs seen late in the infection, as the periosteum becomes more involved. Neonates can present with vague symptoms, consisting only of irritability and poor feeding, or they can show signs of fulminant sepsis. Peripheral WBC count may be normal or elevated with a left shift with a predominance of neutrophils. Erythrocyte sedimentation rate (ESR) is usually elevated and blood cultures are positive in approximately 60% cases. Bone cultures taken either surgically or by needle aspiration result in a culture yield of 80%. Plain radiographs may be normal for up to 2 weeks from the onset of illness, and the earliest signs on plain radiographs are soft-tissue swelling and displacement of muscle plane. Bony changes begin to appear by 7-10 days, starting with a hazy appearance of the metaphysis followed by irregular areas of trabecular necrosis and absorption. Eventually subperiosteal new bone formation occurs as the infection spreads through the cortex. A bone scan usually diagnoses osteomyelitis as early as 24-48 hours from onset. Treatment at first is empiric parenteral antibiotics. The selection of antibiotic should include coverage of Staphylococcus aureus, as well as other organisms; the agent can probably be based on the patient's age and history of illness. Surgical debridement may be necessary if pus is present on needle aspiration or if evidence of either joint involvement or abscess is present. PEARL: Osteomyelitis is an infection of bone that is usually bacterial in origin. Microorganisms can be introduced into bone in three ways: Hematogenous delivery Direct inoculation (usually traumatic, but also surgical) Local invasion from a contiguous infection (usually decubitus ulcer or periodontal disease) In children, acute osteomyelitis is primarily hematogenous in origin. Risk factors for hematogenous osteomyelitis in children include the following: Sickle cell disease Immunodeficiency disorders, such as chronic granulomatous disease Sepsis Minor trauma coincident with bacteremia Indwelling vascular catheters Chronic hemodialysis vascular access The growing metaphyses of long bones are most commonly involved, particularly the femur, tibia, or fibula. Bacteria, principally Staphylococcus aureus, cause the vast majority of cases of acute osteomyelitis in children. Staph aureus is the most common pathogen for children with osteomyelitis. Among patients with sickle cell disease (SCD), hematogenous osteomyelitis is most often caused by Salmonella or other gram-negative organisms, such as E. coli.
A 54-year-old woman is admitted to the hospital for pneumonia. The patient presented to the hospital 2 days ago for cough and fever. She reported temperatures to 38.9° C (102° F) and a cough productive of green, copious sputum. She also reported pleuritic chest pain with deep inspiration. The initial examination revealed diminished breath sounds in the left lower lobe with dullness to percussion, and a chest radiograph revealed a dense left lower lobe infiltrate. Which of the following organisms is most likely responsible for her pneumonia? A. Bordetella pertussis B. Klebsiella pneumoniae C. Mycoplasma pneumoniae D. Pneumococcus E. Staphylococcus aureus
The correct answer is D. The etiology of pneumonia is related to both the age of the patient and the particular risk factors that he or she may exhibit. For patients with no specific risk factors, pneumonia is referred to as community-acquired pneumonia (CAP). CAP has a variable etiology depending on the age of the patient. In patients aged 29-55, the pneumococcus (Streptococcus pneumoniae), a gram-positive organism, is the most frequent agent causing so-called typical or bacterial pneumonia. Bordetella pertussis (choice A) causes whooping cough in children. Most adults in the U. S. have been vaccinated against this organism. However, 20 years after the last booster, immunity begins to fade, and it is reasonably common to see patients aged 55 and older presenting with upper and lower respiratory tract infections caused by this organism. It is therefore now recommended to obtain an adult booster immunization against pertussis. Klebsiella pneumoniae (choice B) is a reasonably frequent source of pneumonia in both hospitalized patients and those with chronic aspiration problems, such as post-stroke patients and alcoholics. Mycoplasma pneumoniae (choice C) is the primary agent responsible for so-called atypical pneumonia in the same age bracket. Staphylococcus aureus (choice E) is a gram-positive organism that causes severe cavitating pneumonia. It is most often responsible for pneumonia in diabetic patients and in patients with influenza pneumonia. PEARL: Streptococcus pneumoniae is the most common cause of pneumonia worldwide. Pulmonary infection with Staph aureus can be seen in patients who also have concomitant infections with influenza (post-influenza pneumonia) and also in the elderly. The most common cause of pneumonia in patients with influenza is still Strep Pneumoniae but those with influenza are more likely to get pneumonia caused by Staph aureus as opposed to the general population.
A 16-year-old boy comes to the health care provider complaining of a persistent sunburn on his face that started when he was on vacation 2 weeks earlier and does not seem to be improving. He also complains of headache, chills, fever, and joint pains that are most prominent in the hands with the pain being the worst in the morning. Aspirin seems to improve the symptoms somewhat. He has no significant past medical history and does not take any medication on a regular basis. He denies allergies to medications. On physical examination, the patient is a well-developed and well-nourished young man. His temperature is 37.3ordmC (99.1ordmF), blood pressure is 140/90 mm Hg, pulse is 80/min, and respirations are 16/min. Inspection of the skin reveals diffuse erythema and edema of the nose and cheeks in a butterfly distribution and erythema and edema of the skin overlying the interphalangeal part of his fingers. The proximal and distal interphalangeal joints of the fingers are edematous and tender and his fingertips are studded with minute violaceous papules tender to touch. On the hard palate, there are several shallow ulcers up to 1 cm in diameter. Laboratory studies show: RBCs 2.8 x 1012/L (normal 4.7-6.1 million cells) WBCs 2.2 x 109/L (normal 5,000-10,000) Platelets 200 x 109/L (normal 150,000-400,000) Erythrocyte sedimentation rate 57 mm/h (normal <20 mm/h) Urine protein 4+ (normal negative) Which of the following is the most appropriate screening lab test for this patient? A. Anticentromere antibodies B. Antihistone antibodies C. Anti-Jo-1 antibodies D. Antinuclear antibody E. Anti-Ro antibodies
The correct answer is D. This patient has systemic lupus erythematosus, an autoimmune inflammatory rheumatoid disease affecting multiple organ systems. The etiology is unknown, but it is believed to be multifactorial. Autoantibodies are produced against self-antigens, especially DNA and other nuclear antigens. Sunlight and some infections can exacerbate episodes of lupus. Antinuclear antibody test is the best screening laboratory study. Antinuclear antibody testing is so sensitive that if the test is negative, a search for another disease should be made. Antibodies to double-stranded DNA are much more specific and are present during active disease. C3, C4, and CH50 are all decreased in active disease. Anti-Smith antibodies do not measure disease activity, but are fairly specific for systemic lupus erythematosus. Renal biopsy confirms lupus nephritis. Anticentromere antibodies (choice A) are sensitive and specific for CREST syndrome, a variant of systemic scleroderma that is characterized by Calcinosis cutis, Raynaud phenomenon, Esophageal dysmotility, Sclerodactyly, and Telangiectasias. Antinuclear antibodies are also positive in 90% of these patients (and nonspecific). Antihistone antibodies (choice B) are formed in drug-induced lupus erythematosus. They are highly specific for this disease and impose a homogenous pattern of antinuclear antibody staining. Procainamide, INH, and hydralazine are the offending agents most commonly encountered as culprits of drug-induced lupus. Anti-Jo-1 antibodies (choice C) are a marker of dermatomyositis with significant muscle involvement. They may be detected in patients who have myositis in the absence of skin disease. Anti-Ro antibodies (choice E) are detected in subacute cutaneous lupus erythematosus, neonatal lupus erythematosus, and Sjoumlgren syndrome. This test can be performed after the antinuclear antibody is positive. PEARL: Systemic lupus erythematosus is an autoimmune disorder that can affect any system in the body. Women of childbearing age are most commonly affected. Antinuclear antibody formation is characteristic of this disorder and this is the initial test that should be performed when this disorder is suspected. ANAs are IgG antibodies. Confirmatory tests include double-stranded DNA and identification of the Smith antigen.
A 37-year-old woman presents with a swollen and tender left wrist. The symptoms began approximately a day earlier and have become increasingly severe. She has noted a fever and shaking chills last night. She has no past medical history and takes no medications, except for ibuprofen for occasional premenstrual cramps. On physical examination the wrist has decreased ROM and is tender to palpation. The joint space has an obvious effusion and is tender, erythematous, and warm. Which of the following is the most appropriate next step in management? ANSWER SELECTION Colchicine Indomethacin IV antibiotics Wrist radiograph Arthrocentesis
The correct answer is E. A monoarticular arthritis should raise the question of a septic versus a crystalline arthritis. Septic arthritis and crystalline arthritis can be notoriously similar in their clinical presentations, so clearly establishing the diagnosis, particularly in a patient with no known history of crystalline disease, is of great advantage before initiating definitive therapy. Crystal-induced and septic-induced joint can coexist in the same joint. An arthrocentesis is the most appropriate choice and allows examination of fluid for WBCs, crystals, and bacteria. Colchicine (choice A) , NSAIDs (choice B) , and antibiotics (choice C) are all inappropriate before the arthrocentesis. If diagnosis of a septic joint is missed, the joint can be destroyed from the bacteria. A radiograph (choice D) would not yield a definitive diagnosis in this situation. PEARL: Any patient who has a hot, swollen, inflamed, and motion-restricted joint should have aspiration performed to help determine the diagnosis. If this joint is a prosthetic joint, this aspiration should be performed in the operating room to perform this aspiration under aseptic conditions. American College of Rheumatology guidelines for a patient with monoarticular arthritis after completion of a thorough history and physical examination: Patients who have a history of significant trauma or focal bone pain should have plain radiographs of the affected joint to rule out fracture, tumor, or metabolic bone disease. In the absence of a history of trauma or following a radiograph which excludes fracture or dislocation, an effusion or other signs of inflammation are markers of infection until proven otherwise. Joint aspiration is therefore the next diagnostic step. A bloody effusion should lead to consideration of a coagulopathy, pseudogout, tumor, trauma, or a Charcot joint; subsequent evaluation includes a PT, PTT, platelet count, and bleeding time. Bone marrow elements present in the synovial fluid are suggestive of an intra-articular fracture. A noninflammatory synovial fluid (e.g., <2,000 WBCs or <75% neutrophils) should lead to consideration of osteoarthritis, soft-tissue injury, or viral infection. Inflammatory joint fluid with crystals establishes the diagnosis of gout or pseudogout. A positive synovial fluid culture establishes the diagnosis of infectious arthritis. A sterile inflammatory joint fluid raises the suspicion of systemic rheumatic disorders; such patients should have further evaluation that may include a CBC, ESR, RF, anti-CCP, liver function tests, HLA-B27, ANA, or Lyme serologies.
A 45-year-old man with history of previously well-controlled hypertension comes in because colleagues at his office are telling him that his appearance is changing. The patient has recently noted that his old shoes felt too tight and he had to buy new, larger shoes. He also complains of shooting pains in his hands which his company nurse told him were consistent with carpal tunnel syndrome. Review of systems reveals increased thirst and urination. His temperature is 37.0°C (98.6°F), blood pressure 190/90 mm Hg, pulse 73/min, and respirations 13/min. Physical examination shows frontal bossing, mandibular enlargement, and widening of the spaces between his teeth. His hands and feet seem larger and more out of proportion to his body size than usual. Which of the following is the most appropriate diagnostic study at this time? A. Cortisol level B. CT scan of brain C. Fasting glucose level D.Growth hormone levels E. Insulin-like growth factor
The correct answer is E. This patient has acromegaly, an excess of growth hormone, evidenced by increased hand and foot size and coarse facial features with a protruding jaw. Other features, such as tooth-space widening and macroglossia, can be present. Impaired glucose tolerance is often present, as is diabetes. Hypertension is present in 1/3 of these patients. Complaints of headache and visual field losses can occur. Patients may have carpal tunnel syndrome. The test of choice for diagnosis is insulin-like growth factor, which is elevated >5 times the normal limit. Another test which can be used for diagnosis is measurement of growth hormone after the administration of 75 g of glucose. In a normal patient, the growth hormone level would be suppressed, but in a patient with acromegaly, the levels remain high (greater than 5 ng/mL). Cortisol levels (choice A) might be helpful as part of the diagnosis of Cushing syndrome. Cushing syndrome can be associated with insulin resistance, hypertension, and weight gain but doesn't share other features associated with acromegaly. CT scan of the brain (choice B) would likely reveal a macroadenoma in this patient. A CT scan (or other imaging) should be done to confirm most endocrine disorders only after they are suggested serologically. This is to avoid confusion of incidentally discovered tumors that are unrelated to the patient condition but can certainly lead one down an incorrect course. Fasting glucose level (choice C) is a useful screening test for diabetes. It is likely that our patient has diabetes secondary to his acromegaly as evidenced by his polyuria and polydipsia. It is appropriate to measure fasting glucose levels in this patient, but it is not the test to diagnose the patient's underlying primary medical problem. Growth hormone levels (choice D) should not be used as a screening test to diagnose acromegaly, because patients have a normal daily fluctuation of gonadotropin-releasing hormone (GNRH) and growth hormone. Growth hormone levels can also fluctuate with exercise and acute illness. Pregnant and adolescent patients can also have increased levels and not have acromegaly. A lack of the normal nadir may be useful in identifying patients who have acromegaly, because these patients will not have full suppression of growth hormone throughout the 24-hour period. PEARL: Acromegaly is a condition in which there is abnormal increased release of growth hormone from the pituitary gland. It is associated with increased morbidity and mortality. Clinical presentations of this condition include increased growth noted in the extremities, carpal tunnel syndrome, arthralgias, glucose intolerance or overt diabetes, hypertension, sleep apnea, and colonic polyp (increased risk for colon cancer as a result). The serum insulin-like growth factor (IGF-1) is increased in the serum, and a random measurement of growth hormone in the serum never falls below 0.4 micrograms. An oral glucose tolerance test may be used to confirm this diagnosis. Trans-sphenoidal surgery may be curative to remove the pituitary adenoma. If surgery fails to correct this problem, the patient may be treated with the somatostatin analog octreotide (Sandostatin) or carbegoline (Dostinex) if prolactin secretion is also part of the abnormal pituitary function.
A 5-year-old boy is brought to the clinic because his parents have noticed swelling around his eyes for the past 3 days. The child was previously well and was last seen for an annual well-child visit 1 month ago when his weight was 18.5 kg (40.8 lb). He has been urinating less than usual but he is otherwise well. No change in the urine color was noticed and there was no fever, diarrhea, or vomiting. Examination reveals an alert and active child. Blood pressure is 98/66 mm Hg. There is periorbital edema as well as pitting edema of the ankles. Lung and abdominal examinations are normal and there is no skin rash. Laboratory studies show: Which of the following is the most appropriate next step in management? ANSWER SELECTION Albumin infusion Cyclophosphamide (Cytoxan) Furosemide (Lasix) High-protein diet Prednisone
The correct answer is E. This patient has nephrotic syndrome, based on the presence of edema, hypoalbuminemia, hyperlipidemia, and proteinuria >3.5 g. Nephrotic syndrome in children is most often idiopathic and common between ages 1 and 8. Most cases are caused by minimal change disease (Nil disease) in which the renal biopsy specimen reveals few or no changes on light microscopy, but under electron microscopy there is effacement of the renal tubular epithelial foot processes. The urine shows heavy protein (>3.5 g) and no abnormal urinary sediment or hematuria. Children ages 1-8 are likely to have steroid-responsive nephrotic syndrome, and thus initial therapy with prednisone is the most appropriate choice. Parents should also be advised to restrict salt intake until the edema subsides. Some children with nephrotic syndrome will develop severe edema with ascites or pleural effusions due to massive proteinuria and require hospitalization. When there is risk for respiratory compromise, albumin infusion (choice A) followed by a potent loop diuretic (such as furosemide, (choice C) may be necessary to mobilize the fluid. In most cases, however, prednisone treatment alone is sufficient to produce resolution of edema. Cyclophosphamide (choice B) is an immunosuppressive drug that is useful for children who suffer from frequent relapses of nephrotic syndrome despite steroid treatment. It is not used as the primary agent for treatment of nephrotic syndrome. A high-protein diet (choice D) has no role in managing the hypoalbuminemia seen in nephrotic syndrome caused by continued large losses of albumin in the urine. It is not first-line therapy. These patients are making large amounts of cholesterol to replace the lost protein and these patients are at risk for hyperlipidemia so increasing protein in the diet would only make this problem worse. PEARL: Nephrotic syndrome is characterized by edema, hypoalbuminemia, proteinuria of >3.5 g, dyslipidemia, and hypercoagulability caused by loss of anti-thrombin III in the urine. Minimal change disease is the most common etiology in children. It is termed such because under light microscopy, the biopsy specimen looks normal, however under electron microscopy, there is effacement of the podocytes (foot processes). Treatment is most commonly steroids, and most children experience a complete remission without recurrence.
A 72-year-old man with a history of obesity and hypertension treated with nifedipine comes to the health care provider complaining of worsening right-sided knee pain. He has suffered from this knee pain for 10 years, but it is no longer relieved with nonsteroidal anti-inflammatory drugs as in the past. The knee pain gets progressively worse as the day goes on and the pain improves with rest. When asked about other joint involvement, he reports that he has less severe pain in his right wrist. His temperature is 37°C (98.6°F), blood pressure 160/90 mm Hg, pulse 68/min, and respirations 15/min. Physical examination reveals an obese man in no acute distress. His knee is not swollen or warm, but crepitus can be appreciated. His left knee and right wrist are normal. Which of the following is consistent with this patient's history? ANSWER SELECTION Antinuclear antibody positive Elevated erythrocyte sedimentation rate Joint erosions on radiograph Rheumatoid factor positive Unequal joint spaces on radiograph
The correct answer is E. This patient has osteoarthritis. Clues to this diagnosis include the patient's risk factors, which include advanced age and obesity. The distribution of joints is also typical, with weight-bearing joints such as the hip and knee being affected together with the small joints of the fingers (DIPs and PIPs). The asymmetric involvement is common. Morning stiffness is seen but resolves in a short period of time, as opposed to that of rheumatoid arthritis. The more typical complaint is worsening joint pain with activity. Crepitus can be appreciated on physical examination of the involved joint. The common radiographic finding is unequal joint spaces with osteophyte formation. Laboratory results are typically normal. Antinuclear antibodies (choice A) are not likely to be positive in this patient. It is best not to do antinuclear antibody testing in this patient, because it can be positive in up to 10% of patients who do not have any rheumatologic disease. A false positive result, therefore, might be confusing. Elevated erythrocyte sedimentation rate (choice B) and C-reactive protein, which are markers of inflammation, are not seen in the setting of osteoarthritis. Joint erosions (choice C) are not seen with osteoarthritis. Rather, they are seen in rheumatoid arthritis, and if found, may be helpful in confirming that diagnosis. Rheumatoid factor (choice D) would also be negative. Rheumatoid arthritis is more commonly symmetric in its joint involvement. This patient's history is not consistent with rheumatoid arthritis. PEARL: Osteoarthritis is common in older, obese patients and involves weight-bearing joints, such as the knee and hip. Osteophytes and uneven joint spaces are characteristic on radiograph. A common physical finding is crepitus, described as a "crunching" feeling on palpation with simultaneous movement of the joint.
A 38-year-old woman comes to the office complaining of a new-onset rash on her face, hair loss, malaise, and fatigue that has been worsening over the previous month. She has also noticed that sun exposure makes her rash flare to a great degree. Her medical history is significant for food allergies and acute thyroiditis as a teenager. Her only medication is a daily thyroid hormone supplement. On physical examination, there is ill-defined erythema and edema on the nose and cheeks. There is diffuse thinning of the scalp hair, with short hairs on the anterior hairline. The fingertips are studded with minute, violaceous papules that are tender to touch. Urinalysis shows 4+ protein. Laboratory studies show: Red blood cells2.9 x 1012/LWhite blood cells2.0 x 109/LPlatelets220 x 109/LErythrocyte sedimentation rate57 mm/hAnti-nuclear antibody1:160 Which of the following antibodies is the most specific serologic test for this disease? ANSWER SELECTION A. Anticentromere antibodies B. Antihistone antibodies C. Anti-Jo-1 antibodies D. Anti-Ro antibodies E. Anti-Smith antibodies
The correct answer is E. Young to middle-aged women are predominantly affected with systemic lupus erythematosus, showing a wide range of signs and symptoms. Skin involvement occurs in 80% of cases and is likely to be very helpful in making the correct diagnosis. Of all the serologic tests, anti-Smith antibodies carry the highest specificity for lupus erythematosus. Anticentromere antibodies (choice A) are sensitive and specific for CREST syndrome, a variant of systemic scleroderma that is characterized by calcinosis cutis, Raynaud phenomenon, esophageal dysmotility, sclerodactyly, and telangiectasias. Antinuclear antibodies are also positive in 90% of these patients. Due to their high degree of sensitivity, antinuclear antibodies are used as the screening test for lupus. Antihistone antibodies (choice B) are formed in drug-induced lupus erythematosus. They are highly specific for this disease and impose a homogenous pattern of antinuclear antibody staining. Procainamide, methyldopa, and hydralazine are the offending agents most commonly encountered as culprits of drug-induced lupus. Anti-Jo-1 antibodies (choice C) are a marker of dermatomyositis with significant muscle involvement. They can also be detected in patients who have myositis in the absence of skin disease. Anti-Ro antibodies (choice D) are detected in subacute cutaneous lupus erythematosus, neonatal lupus erythematosus, and Sjogren syndrome, and should be valued in the context of clinicopathologic correlation. PEARL: The screening test for SLE is the antinuclear antibody, which is seen in practically all patients who have SLE. Specific tests that are used as confirmatory tests include double-stranded DNA and anti-Smith antibodies.
Two bewildered parents rush into the emergency department carrying a 3-year-old girl. They tell the health care provider at the door that she ingested multiple acetaminophen pills less than an hour ago. The mother was experiencing a terrible headache over the past 2 days and was taking acetaminophen as a pain reliever. She left the open bottle on her desk to get a glass of water, and when she returned her daughter was putting the tablets in her mouth. Immediately the mother removed whatever pills were still in the child's mouth and checked to insure that all the other medications were safely stored away; when she saw that they were untouched in the medicine cabinet, the family got in the car to rush to the hospital. On route there the girl started vomiting. The mother does not know how many pills her daughter took because it wasn't a full bottle to begin with. At the hospital she takes out the half-empty bottle and spills the contents on the counter. A total of 36 pills is present from an original 100 count, 325 mg of acetaminophen each. The child, crying on the examining table, appears frightened and diaphoretic, and repeats several times that she wants to throw up. Her pulse is 120/min and respirations are 24/min. Which of the following is the most appropriate next step? A. Administration of dextrose and naloxone (Narcan) B. Admistration of N-acetylcysteine (Mucomyst, Acetadote) intravenously C.Administration of N-acetylcysteine orally D. Immediate measurement of plasma concentration of acetaminophen E. Supportive care and measurement of plasma concentration of acetaminophen 3 hours later
he correct answer is E. Acetaminophen is an analgesic and antipyretic that is metabolized in the liver. Acetaminophen poisoning causes an N-acetyl-p-benzoquinoneimine metabolite that produces hepatotoxicity in the absence or depletion of glutathione. In children younger than age 12 the toxic dose is 150 mg/kg. A single ingestion of 7.5 g is considered to be a minimum toxic dose in adolescents and adults. There are 4 stages of acetaminophen poisoning if the patient is left untreated. Absorption may be delayed 4 hours in an overdose. A plasma level of acetaminophen therefore should be measured until it has been at least 4 hours since the ingestion. Levels drawn before that may not be accurate and may give a false low result. The Rumack-Matthew nomogram is used to plot the level of acetaminophen based on the time elapsed since ingestion and the acetaminophen plasma concentration. N-acetylcysteine is the antidote for acetaminophen poisoning, and it should be administered if the acetaminophen level is equal to or greater than 150 µg/ml. N-acetylcysteine works best if started within 8 hours of ingestion, but it may be given up to 16 hours after ingestion. In cases of severe poisoning, Poison Control may advise N-acetylcysteine be given as late as 24 to 36 hours after ingestion has occurred. In the United States, N-acetylcysteine is given orally. Liver enzymes, bilirubin levels, and prothrombin time should be monitored in patients whose plasma levels of acetaminophen are within the toxic range in the nomogram. In patients who have fulminant hepatic necrosis, death may occur. Severe hepatic damage may necessitate a liver transplant in some patients. Dextrose and naloxone administration (choice A) is indicated in patients who present with an altered mental status or respiratory depression following narcotic and opioid ingestions. Because there are no such signs in this patient and because it is suspected that the child ingested only one drug, acetaminophen, dextrose and nalaxone (Narcan) is not indicated at this time. It is not indicated to administer N-acetylcysteine intravenously (choice B) , because the acetaminophen level has not yet been determined and so there are no data as yet indicating that it is necessary. N-acetylcysteine is typically given orally, but there is an alternative intravenous delivery system that can also be given via protocol. Administration of N-acetylcysteine orally (choice C) may be indicated once a toxic plasma level of acetaminophen is confirmed. That is not the case here, however. Also, the symptoms that the child is exhibiting do not necessarily have to be indicative of acetaminophen poisoning, but may be the result of a sympathicomimetic response to a frightening situation. The administration of N-acetylcysteine therefore should be delayed until data are available to support its necessity. Absorption of acetaminophen may be delayed 4 hours in an overdose. A plasma level of acetaminophen therefore should be measured when it has been at least 4 hours since the ingestion. Levels drawn before that time may not be accurate and may give a false low result. Immediately measuring plasma concentration of acetaminophen (choice D) is therefore not indicated.